Wrong three digits

This problem is written as a continuation of this problem .

Brian was bored again. He wanted to know if he could find a fraction that had an orderly pattern after decimal such that the pattern would show progressively increasing square numbers.

He then tried to analyze the number 1 999 + 3 ( 999 × 999 ) + 2 ( 999 × 999 × 999 ) = 1 001 000 997 002 999 \frac{1}{999} + \frac{3}{(999 \times 999)} + \frac{2}{(999 \times 999 \times 999)} = \frac{\num{1001000}}{\num{997002999}}

He found quite a satisfying fraction which shows 0.001 004 009 016 025 036 049 , \num{0.001004009016025036049} \ldots, but he knew that the pattern would end since infinitely many square numbers were larger than 1000. However, he believed that he could predict which number was placed in each digit. Now Brian wanted to know what number was in the 409 th ^\text{th} , 410 th ^\text{th} , and 411 th ^\text{th} digit after the decimal point. Submit your answer by concatenating the three digits in order.


If you like the problem, the next problem in this series is: C o n s t r u c t i n g f o u r d i g i t s \href{https://brilliant.org/problems/constructing-four-digits-brian-is-bored-again/}{Constructing \ four \ digits}


The answer is 788.

This section requires Javascript.
You are seeing this because something didn't load right. We suggest you, (a) try refreshing the page, (b) enabling javascript if it is disabled on your browser and, finally, (c) loading the non-javascript version of this page . We're sorry about the hassle.

4 solutions

Afkar Aulia
Apr 26, 2021

It's helpful to see the fractions in decimals. We can also change the fractions into sequences:

1 999 \frac{1}{999} = 0.001001001001001001....

Since x 999 \frac{x}{999} = x 1000 \frac{x}{1000} + x ( 1000 × 999 ) \frac{x}{(1000 \times 999)} , we can get..

3 ( 999 × 999 ) \frac{3}{(999 \times 999)} = 3 ( 999 × 1000 ) \frac{3}{(999 \times 1000)} + 3 ( 999 × 999 × 1000 ) \frac{3}{(999 \times 999 \times 1000)} = 3 ( 999 × 1000 ) \frac{3}{(999 \times 1000)} + 3 ( 999 × 1000 × 1000 ) \frac{3}{(999 \times 1000 \times 1000)} + 3 ( 999 × 999 × 1000 × 1000 ) \frac{3}{(999 \times 999 \times 1000 \times 1000)} =

3 ( 999 × 1000 ) \frac{3}{(999 \times 1000)} + 3 ( 999 × 1000 × 1000 ) \frac{3}{(999 \times 1000 \times 1000)} + 3 ( 999 × 1000 × 1000 × 1000 ) \frac{3}{(999 \times 1000 \times 1000 \times 1000)} + 3 ( 999 × 1000 × 1000 × 1000 × 1000 ) \frac{3}{(999 \times 1000 \times 1000 \times 1000 \times 1000)} + .....

Since 3 999 \frac{3}{999} = 0.003003003003003003... then 3 ( 999 × 999 ) \frac{3}{(999 \times 999 )} = 0.000003006009012015018...

With the same way, we can see that 2 ( 999 × 999 × 999 ) \frac{2}{(999 \times 999 \times 999)} = 1 1000 \frac{1}{1000} X 2 ( 999 × 999 ) \frac{2}{(999 \times 999 )} + 1 1000 \frac{1}{1000} X 1 1000 \frac{1}{1000} X 2 ( 999 × 999 ) \frac{2}{(999 \times 999 )} + 1 1000 \frac{1}{1000} X 1 1000 \frac{1}{1000} X 1 1000 \frac{1}{1000} X 2 ( 999 × 999 ) \frac{2}{(999 \times 999 )} + ... =

0.000000002004006008010012014.. +

0.000000000002004006008010012014.. +

0.000000000000002004006008010012014.. +

0.000000000000000002004006008010012014.. + .....

Now we know that the first three digits of 2 ( 999 × 999 × 999 ) \frac{2}{(999 \times 999 \times 999)} is 000, the second is 000, the third is 002, the 4th is 006 or 002+004, the 5th is 012 or 002+004+006 , so now we can see that starting from the third three-digited number until a certain digits, the nth 3-digited number after decimal follows the arithmetic sequence of (n-2)(n-1) The following digits should still follow the pattern with the mistakes caused by the resulting sum being larger than 1000, and thus, give the excess to the 3-digit preceding it.

Now we can see that the sum of the three fraction caused the nth 3-digited number after decimal to follow the formula of: 1 + 3 (n-1) + (n-2)(n-1) = n 2 n^{2}

This method of predicting digit becomes wrong after n >= 31, such that the next n satisfies n 2 n^{2} > 1000 such that the three digited number starting from digit 94th, 95th, and 96th, gives the excess sum to the digit 93rd, causing the 31st three-digited number becomes 962 instead.

With this logic, the 409th, 410th, and 411th digit is the last three digit of 13 7 2 137^{2} and the excess given by the sum of the following three digits, which is 13 8 2 138^{2}

13 7 2 137^{2} = 18769

13 8 2 138^{2} = 19044

And thus, the 409th, 410th, and 411th digit, when concatenated, has the value of 769 + 19 = 788

Oximas Omar
May 9, 2021

position 411 411 contains the square of 411 3 = 137 \frac{411}{3} = 137 , 13 7 2 = 18769 137^2=18769 ,this concatinates with the first 2 digits of next number 13 8 2 = 19044 138^2=19044

so taking the numbers at positions 409,410,411 we get 769 769 from 18769 18769 and 19 19 from 19044 19044 wich add to 769 + 19 = 788 769+19=788

Carsten Meyer
May 3, 2021

For simplicity, let q = 1 0 3 q=10^{-3} . Then we can do partial fraction decomposition (PFD) to reorder Brian's new number X X into generalized geometric series ( ) (*) : X = q 1 q + 3 q 2 ( 1 q ) 2 + 2 q 3 ( 1 q ) 3 = q 2 + q ( 1 q ) 3 = PFD 1 ( 1 q ) 3 ( 1 q ) 2 + 2 ( 1 q ) 3 = ( ) k = 0 [ ( k + 0 0 ) 3 ( k + 1 1 ) + 2 ( k + 2 2 ) ] q k = k = 0 [ 1 3 ( k + 1 ) + ( k + 1 ) ( k + 2 ) ] q k = k = 0 k 2 q k = : k = 0 a k \begin{aligned} X &= \frac{q}{1-q} + \frac{3q^2}{(1-q)^2} + \frac{2q^3}{(1-q)^3} = \frac{ q^2+q }{(1-q)^3} \underset{\text{PFD}}{=} \frac{1}{(1-q)} - \frac{3}{(1-q)^2} + \frac{2}{(1-q)^3}\\[1em] &\underset{(*)}{=}\sum_{k=0}^\infty\left[ \binom{k+0}{0}-3\binom{k+1}{1}+2\binom{k+2}{2} \right]q^k=\sum_{k=0}^\infty [1 -3(k+1) + (k+1)(k+2)]q^k = \sum_{k=0}^\infty k^2q^k=:\sum_{k=0}^\infty a_k \end{aligned} The only elements that can influence the digits d 409 ; d 410 ; d 411 d_{409};\:d_{410};\:d_{411} are a 137 a_{137} and a 138 a_{138} . Let's write down their digits: a 137 = 18769 1 0 411 , a 138 = 19044 1 0 414 k 407 408 409 410 411 412 413 414 a 137 1 8 7 6 9 a 138 1 9 0 4 4 7 8 8 0 \begin{aligned}a_{137} &= 18769\cdot 10^{-411}, & a_{138} &= 19044\cdot 10^{-414} &&&\Rightarrow &&&& \begin{array}{r|rrrrrrrr} k & 407 & 408 & 409 & 410 & 411 & 412 & 413 & 414\\\hline a_{137} & 1 & 8 & 7 & 6 & 9 & &\\ a_{138} & & & & 1 & 9 & 0 & 4 & 4\\\hline & & & \red{7} & \red{8} & \red{8} & 0 & & \end{array}\end{aligned} Assuming the remainder of the series stays small enough that it does not influence these digits, the answer are the red digits 788 \boxed{788}


The generalized geometric series:

k = 0 ( k + n k ) q k = 1 ( 1 q ) n + 1 , q < 1 , n N ( ) \begin{aligned} \sum_{k=0}^\infty \binom{k+n}{k}q^k&=\frac{1}{(1-q)^{n+1}},&&&|q|&<1, &n&\in\mathbb{N}&(*) \end{aligned}

Saya Suka
Apr 29, 2021

Answer
= (411 ÷ 3)² mod 10³ + floor{ [(411 ÷ 3) + 1]² / 10³ }
= 137² mod 10³ + floor{ 138² / 10³ }
= 18769 mod 1000 + floor{ 19044 / 1000 }
= 769 + 19
= 788




Lucky 8, 88.8889%

0 pending reports

×

Problem Loading...

Note Loading...

Set Loading...